Difference between revisions of "2011 AMC 12A Problems/Problem 7"

(Solution)
(Solution 3 (essentially Solution 1 and 2 but cleaned with variables))
 
(20 intermediate revisions by 11 users not shown)
Line 1: Line 1:
 
== Problem ==
 
== Problem ==
A majority of the <math>30</math> students in Ms. Demeanor's class bought pencils at the school bookstore. Each of these students bought the same number of pencils, and this number was greater than <math>1</math>. The cost of a pencil in cents was greater than the number of pencils each student bought, and the total cost of all the pencils was <math>17.71</math>. What was the cost of a pencil in cents?
+
A majority of the <math>30</math> students in Ms. Demeanor's class bought pencils at the school bookstore. Each of these students bought the same number of pencils, and this number was greater than <math>1</math>. The cost of a pencil in cents was greater than the number of pencils each student bought, and the total cost of all the pencils was <math>\$17.71</math>. What was the cost of a pencil in cents?
  
 
<math>
 
<math>
Line 9: Line 9:
 
\textbf{(E)}\ 77 </math>
 
\textbf{(E)}\ 77 </math>
  
== Solution ==
+
== Solution 1 ==
<cmath>1771 = 7 \times 11 \times 23</cmath>
+
The total cost of the pencils can be found by <math>(\text{students}\cdot\text{pencils purchased by each}\cdot\text{price of each pencil})</math>.
  
The only factor of 1771 between 16 and 30 (16 because it has to be a majority of her students) is 23. Therefore, we can conclude that there are 23 students, 11 cents per pencil, and each student bought 7 pencils. <math>\Rightarrow \boxed{B}</math>
+
Since <math>1771</math> is the product of three sets of values, we can begin with prime factorization, since it gives some insight into the values: <math>7, 11, 23</math>. Since neither <math>(C)</math> nor <math>(E)</math> are any of these factors, they can be eliminated immediately, leaving <math>(A)</math>, <math>(B)</math>, and <math>(D)</math>.
 +
 
 +
Beginning with <math>(A) 7</math>, we see that the number of pencils purchased by each student must be either <math>11</math> or <math>23</math>. However, the problem states that the price of each pencil must exceed the number of pencils purchased, so we can eliminate this.
 +
 
 +
Continuing with <math>(B) 11</math>, we can conclude that the only case that fulfils the restrictions are that there are <math>23</math> students who each purchased <math>7</math> such pencils, so the answer is <math>\boxed{B}</math>. We can apply the same logic to <math>(E)</math> as we applied to <math>(A)</math> if one wants to make doubly sure.
 +
 
 +
== Solution 2 ==
 +
We know the total cost of the pencils can be found by <math>(\text{students}\cdot\text{pencils purchased by each}\cdot\text{price of each pencil})</math>.
 +
 
 +
Using prime factorization like in the solution above, we see that there is only one combination of three whole numbers whose product is equal to <math>1771</math>: <math>7, 11, 23</math> (without using <math>1</math>). So we know that <math>7, 11</math>, and <math>23</math> must be the number of students, the number of pencils purchased by each student and the price of each pencil in cents.
 +
 
 +
We know that <math>23</math> must be the number of students, as it is the only number that makes up the majority of 30.
 +
 
 +
We pick the greater of the remaining numbers for the price of each pencil in cents, which is <math>11</math>.
 +
 
 +
Therefore, our answer is <math>\boxed{B}</math>.
 +
 
 +
-JinhoK
 +
 
 +
==Solution 3 (essentially Solutions 1 and 2 but cleaned with variables)==
 +
 
 +
We let <math>s</math> be the number of students that bought pencils at the bookstore, <math>c</math> be the cost of each pencil in cents, and <math>n</math> the number of pencils each student bought. Thus, we are looking for <math>c.</math>
 +
Since a majority of the students in the class bought pencils at the bookstore, <math>s>\dfrac{30}2=15.</math> (s>15) We also know that <math>n>1</math> and <math>c>n>1.</math> Finally, <math>s\cdot c\cdot n=1771.</math> We can factor <math>1771</math> as <math>7\cdot11\cdot23.</math> Since <math>n>15</math> and <math>c>n>1,</math> none of them can be <math>1,</math> and therefore <math>c,n,</math> and <math>s</math> are <math>7,11,</math> and <math>23</math> in some order. We know that <math>n>15,</math> so <math>n</math> must be <math>23.</math> <math>c>n,</math> so <math>c=11</math> and <math>n=7.</math> Thus, <math>c=\boxed{\text{(B)}~11}</math> cents.
 +
~Technodoggo
 +
 
 +
==Video Solution==
 +
 
 +
https://www.youtube.com/watch?v=6tlqpAcmbz4
 +
~Shreyas S
 +
 
 +
https://youtu.be/E5UoNyFjSO0
 +
 
 +
~savannahsolver
  
 
== See also ==
 
== See also ==
 
{{AMC12 box|year=2011|num-b=6|num-a=8|ab=A}}
 
{{AMC12 box|year=2011|num-b=6|num-a=8|ab=A}}
 +
{{AMC10 box|year=2011|num-b=9|num-a=11|ab=A}}
 +
{{MAA Notice}}

Latest revision as of 19:13, 10 August 2023

Problem

A majority of the $30$ students in Ms. Demeanor's class bought pencils at the school bookstore. Each of these students bought the same number of pencils, and this number was greater than $1$. The cost of a pencil in cents was greater than the number of pencils each student bought, and the total cost of all the pencils was $$17.71$. What was the cost of a pencil in cents?

$\textbf{(A)}\ 7 \qquad \textbf{(B)}\ 11 \qquad \textbf{(C)}\ 17 \qquad \textbf{(D)}\ 23 \qquad \textbf{(E)}\ 77$

Solution 1

The total cost of the pencils can be found by $(\text{students}\cdot\text{pencils purchased by each}\cdot\text{price of each pencil})$.

Since $1771$ is the product of three sets of values, we can begin with prime factorization, since it gives some insight into the values: $7, 11, 23$. Since neither $(C)$ nor $(E)$ are any of these factors, they can be eliminated immediately, leaving $(A)$, $(B)$, and $(D)$.

Beginning with $(A) 7$, we see that the number of pencils purchased by each student must be either $11$ or $23$. However, the problem states that the price of each pencil must exceed the number of pencils purchased, so we can eliminate this.

Continuing with $(B) 11$, we can conclude that the only case that fulfils the restrictions are that there are $23$ students who each purchased $7$ such pencils, so the answer is $\boxed{B}$. We can apply the same logic to $(E)$ as we applied to $(A)$ if one wants to make doubly sure.

Solution 2

We know the total cost of the pencils can be found by $(\text{students}\cdot\text{pencils purchased by each}\cdot\text{price of each pencil})$.

Using prime factorization like in the solution above, we see that there is only one combination of three whole numbers whose product is equal to $1771$: $7, 11, 23$ (without using $1$). So we know that $7, 11$, and $23$ must be the number of students, the number of pencils purchased by each student and the price of each pencil in cents.

We know that $23$ must be the number of students, as it is the only number that makes up the majority of 30.

We pick the greater of the remaining numbers for the price of each pencil in cents, which is $11$.

Therefore, our answer is $\boxed{B}$.

-JinhoK

Solution 3 (essentially Solutions 1 and 2 but cleaned with variables)

We let $s$ be the number of students that bought pencils at the bookstore, $c$ be the cost of each pencil in cents, and $n$ the number of pencils each student bought. Thus, we are looking for $c.$ Since a majority of the students in the class bought pencils at the bookstore, $s>\dfrac{30}2=15.$ (s>15) We also know that $n>1$ and $c>n>1.$ Finally, $s\cdot c\cdot n=1771.$ We can factor $1771$ as $7\cdot11\cdot23.$ Since $n>15$ and $c>n>1,$ none of them can be $1,$ and therefore $c,n,$ and $s$ are $7,11,$ and $23$ in some order. We know that $n>15,$ so $n$ must be $23.$ $c>n,$ so $c=11$ and $n=7.$ Thus, $c=\boxed{\text{(B)}~11}$ cents. ~Technodoggo

Video Solution

https://www.youtube.com/watch?v=6tlqpAcmbz4 ~Shreyas S

https://youtu.be/E5UoNyFjSO0

~savannahsolver

See also

2011 AMC 12A (ProblemsAnswer KeyResources)
Preceded by
Problem 6
Followed by
Problem 8
1 2 3 4 5 6 7 8 9 10 11 12 13 14 15 16 17 18 19 20 21 22 23 24 25
All AMC 12 Problems and Solutions
2011 AMC 10A (ProblemsAnswer KeyResources)
Preceded by
Problem 9
Followed by
Problem 11
1 2 3 4 5 6 7 8 9 10 11 12 13 14 15 16 17 18 19 20 21 22 23 24 25
All AMC 10 Problems and Solutions

The problems on this page are copyrighted by the Mathematical Association of America's American Mathematics Competitions. AMC logo.png